Which equation can be simplified to find the inverse of y = 2x2

Which Equation Can Be Simplified To Find The Inverse Of Y = 2x2

Answers

Answer 1

Answer: d

Step-by-step explanation:

edge 2020


Related Questions

Suppose that g(x)= f(x)+ 6. Which statement best compares the graph of g(x) with the graph of f(x)?

A. The graph of g(x) is the graph of f(x) shifted 6 units down.

B. The graph of g(x) is the graph of f(x) shifted to the right.

C. The graph of g(x) is the graph of f(x) shifted 6 units to the left.

D. The graph of g(x) is the graph of f(x) shifted 6 units up.

Answers

Answer:

D

Step-by-step explanation:

The + 6 moves it up 6 units.

The correct answer is (D) "The graph of g(x) is the graph of f(x) shifted 6 units up."

What is the function?

A relationship between a group of inputs and one output is referred to as a function. In plain English, a function is an association between inputs in which each input is connected to precisely one output. A domain, codomain, or range exists for every function. Typically, f(x), where x is the input, is used to represent a function.

When we add a constant to a function, such as in the case of g(x) = f(x) + 6, it will shift the graph of f(x) upward by 6 units.

This is because, for any value of x, the value of f(x) will be added to 6, resulting in a vertical shift of the entire graph.

Option (A) is incorrect because adding 6 to f(x) would shift the graph up, not down.

Option (B) is incorrect because adding a constant to a function does not cause it to shift horizontally.

Option (C) is incorrect because adding 6 to f(x) would shift the graph right, not left.

D. The graph of g(x) is the graph of f(x) shifted 6 units up. Adding a constant term to a function will shift the graph of the function vertically. In this case, adding 6 to f(x) will shift the graph of f(x) upward by 6 units, resulting in the graph of g(x).

Learn more about function here:

https://brainly.com/question/29633660

#SPJ7

Rectangle ABCD translates 4 units down and 2 units to the right to form rectangle A'B'C'D'. The vertices of rectangle ABCD are labeled in alphabetical order going clockwise around the figure. If AB = 3 units and AD = 5 units, what is the length of B'C'?

Answers

Answer:

The length of BC is 14 units.

Step-by-step explanation:

[tex]hope \: \: it \: \: helps} \beta \alpha \infty [/tex]

The length of B'C' is 0 units.

What is translation?

It is the movement of the shape in left, right, up, and down direction.

The translated shape will have the same shape and shape.

There is a positive value when translated to the right and up.

There is a negative value when translated to the left and down.

We have,

The length of AD = 5 units.

Since the rectangle translates down by 4 units,

The length of A'D' =5 units.

The width of the original rectangle is AB, which is 3 units.

Since the rectangle translates to the right by 2 units,

The width of the new rectangle = 3 units.

Now,

The length of B'C' is the same as the length of AD', which is 5 units.

Subtracting 5 units from 5 units gives us a length of 0 units.

Thus,

The length of B'C' is 0 units.

Learn more about translation here:

https://brainly.com/question/12463306

#SPJ7

0.7(1.5 + y) = 3.5y - 1.47

Answers

Answer:

y = 0.9

Step-by-step explanation:

1.05 + 0.7y = 3.5y - 1.47

-3.5y + 0.7y = -1.47 - 1.05

-2.8y = -2.52

y = 9/10 = 0.9

Answer:

[tex]\textbf{HELLO!!}[/tex]

[tex]0.7\left(1.5+y\right)=3.5y-1.47[/tex]

[tex]1.05+0.7y=3.5y-1.47 \gets \textsl{Expand}[/tex]

[tex]1.05+0.7y-1.05=3.5y-1.47-1.05 \gets Subtract\; 1.05 \from\:both\:sides[/tex]

[tex]0.7y=3.5y-2.52[/tex]

[tex]0.7y-3.5y=3.5y-2.52-3.5y[/tex]

[tex]\mathrm{Subtract\:}3.5y\mathrm{\:from\:both\:sides} \nwarrow[/tex]

[tex]-2.8y=-2.52[/tex]

[tex]\frac{-2.8y}{-2.8}=\frac{-2.52}{-2.8} \hookleftarrow \mathrm{Divide\:both\:sides\:by\:}-2.8[/tex]

[tex]\boxed{\boxed{\underline{\textsf{\textbf{y=0.9}}}}}[/tex]

[tex]\bullet \bullet \bullet \bullet \bullet \bullet \bullet \bullet \bullet \bullet \bullet \bullet \bullet[/tex]

[tex]\textbf{HOPE IT HELPS}[/tex]

[tex]\textbf{HAVE A GREAT DAY!!}[/tex]

Six consecutive numbers add up to a total of 69.what is the highest of these numbers?

Answers

Answer:

14

Step-by-step explanation:

9+10+11+12+13+14=69

The [tex]HIGHEST[/tex] of these numbers is [tex]14[/tex]

Let the [tex]CONSECUTIVE[/tex] numbers be : [tex]a, a+1, a +2,a+3,a+4,a+5[/tex]

Taking the sum :

[tex]a + a + 1 + a + 2 + a + 3 + a + 4 + a + 5 = 69\\6a + 15 = 69\\6a = 69 - 15\\6a = 54\\a = 54 / 6\\a = 9[/tex]

[tex]HIGHEST[/tex] value = [tex]a + 5 = 9 + 5 = 14[/tex]

Hence, [tex]HIGHEST[/tex] value = [tex]14[/tex]

Learn more : https://brainly.com/question/15974141

A road has a scale of 1:50 000 The length of a road on the map is 8.5cm.Work out the length of the real road in kilometres

Answers

Answer:

ok so

8.5*150000

1275000 cm into kilometers is

12.75 kilometers

Hope This Helps!!!

Need help finding the factor of 2y^2-2y-4

Answers

Answer:

hope it helps you............

Answer:

2(y - 2)(y + 1)

Step-by-step explanation:

Given

2y² - 2y - 4 ← factor out 2 from each term

= 2(y² - y - 2) ← factor the quadratic

Consider the factors of the constant term (- 2) which sum to give the coefficient of the y- term (- 1)

The factors are - 2 and + 1, since

- 2 × 1 = - 2 and - 2 + 1 = - 1 , then

y² - y - 2 = (y - 2)(y + 1)

Then

2y² - 2y - 4 = 2(y - 2)(y + 1) ← in factored form

the sum of two consecutive numbers is 2x+3. What are the numbers

Answers

Answer: 2 and 3

Step-by-step explanation:

its numbers

What is the value of the expression 3m-4.2 If m equals 2.1

Answers

Answer:

2.1

Step-by-step explanation:

Given :

3m-4.2 where, m=2.1

Now,

3(2.1)-4.2

6.3-4.2

2.1

Answer is 2.1

Can someone answer with steps and explanation? Thanks.

Answers

Answer:

[tex]x=-16\text{ or } x=7[/tex]

Step-by-step explanation:

Since ΔABC is mapped onto ΔDEF, we can write that:

[tex]\Delta ABC\cong \Delta DE F[/tex]

By CPCTC:

[tex]\angle A\cong \angle D[/tex]

And since ΔABC is isosceles with Vertex C:

[tex]\angle A \cong \angle B[/tex]

We are given that:

[tex]m\angle D=34[/tex]

Hence:

[tex]m\angle A=34=m\angle B[/tex]

We are also given that:

[tex]m\angle C=x^2+9x[/tex]

The interior angles of a triangle must sum to 180°. Thus:

[tex]m\angle A+m\angle B+m\angle C=180[/tex]

Substitute:

[tex](34)+(34)+(x^2+9x)=180[/tex]

Simplify:

[tex]68+x^2+9x=180[/tex]

Isolate the equation:

[tex]x^2+9x-112=0[/tex]

Factor:

[tex](x+16)(x-7)=0[/tex]

Zero Product Property:

[tex]x+16=0\text{ or } x-7=0[/tex]

Solve for each case:

[tex]x=-16\text{ or } x=7[/tex]

Testing the solutions, we can see that both yields C = 112°.

Hence, our solutions are:

[tex]x=-16\text{ or } x=7[/tex]

Chung has 6 trucks and 5 cars in his toy box. Brian has 4 trucks and 5 cars in his toy box.

Which is the correct comparison of their ratios of trucks to cars?
StartFraction 6 Over 4 EndFraction less-than StartFraction 5 Over 5 EndFraction
StartFraction 6 Over 4 EndFraction greater-than StartFraction 5 Over 5 EndFraction
StartFraction 6 Over 5 EndFraction less-than StartFraction 4 Over 5 EndFraction
StartFraction 6 Over 5 EndFraction greater-than StartFraction 4 Over 5 EndFraction

Answers

Given:

Chung has 6 trucks and 5 cars in his toy box.

Brian has 4 trucks and 5 cars in his toy box.

To find:

The correct comparison of their ratios of trucks to cars.

Solution:

The ratio of trucks to cars is defined as:

[tex]\text{Ratio}=\dfrac{\text{Number of trucks}}{\text{Number of cars}}[/tex]

Chung has 6 trucks and 5 cars in his toy box. So, the ratio of trucks to cars is:

[tex]\text{Ratio}=\dfrac{6}{5}[/tex]

Brian has 4 trucks and 5 cars in his toy box.

[tex]\text{Ratio}=\dfrac{4}{5}[/tex]

We know that,

[tex]6>4[/tex]

[tex]\dfrac{6}{5}>\dfrac{4}{5}[/tex]

Therefore, the correct option is D.

Answer:

what the guy above me said

Step-by-step explanation:

so yeah he is right points

find the area of the following figures: ​

Answers

Answer:

Area = 156 square cm

Step-by-step explanation:

Alec bakes spherical rolls of bread. Each roll is about 8cm
wide. What is the approximate volume of each roll? Use
3.14 to approximate a.

Answers

Answer:

Step-by-step explanation:

2143.57

ASAP there are three marbles in a bag. One is red and two are black. What is the probability of picking a black marble first, putting it back in the bag and then picking a black marble? Use the following probability to find the answer.

Answers

Answer:

[tex] \frac{4}{9} [/tex]

Step-by-step explanation:

[tex]p = \frac{favorable \: outcomes}{total \: outcomes} = \frac{4}{9} [/tex]

Answer:  Choice D)  4/9

=============================================================

Explanation:

The probability you get a black marble on the first selection is 2/3 since we have 2 black marbles out of 2+1 = 3 total.

We put the marble back and then we have 2/3 as the probability of selecting another black marble on the second try. Nothing has changed because we put the marble back. That means the events are independent.

So we get (2/3)*(2/3) = 4/9 as the probability of selecting 2 black marbles in a row (with replacement).

what is 5.73 as a mixed number​

Answers

Answer:

[tex]5 \ \frac{73}{100} [/tex]

Step-by-step explanation:

Convert the decimal number to a fraction by placing the decimal number over a power of ten. Since there are

2 numbers to the right of the decimal point, place the decimal number over 10^2 (100). Next, add the whole number to the left of the decimal.

Answer:

5 73/100

Step-by-step explanation:

5.73 = 573

100

= 573

100

as a fraction

To convert the decimal 5.73 to a fraction, just follow these steps:

Step 1: Write down the number as a fraction of one:

5.73 = 5.73

1

Step 2: Multiply both top and bottom by 10 for every number after the decimal point:

As we have 2 numbers after the decimal point, we multiply both numerator and denominator by 100. So,

5.73

1

= (5.73 × 100)

(1 × 100)

= 573

100

.

(This fraction is alread reduced,

As the numerator is greater than the denominator, we have an IMPROPER fraction, so we can also express it as a MIXED NUMBER, thus 573

100

is also equal to 5 73/100

when expressed as a mixed number.

Moving to another question will save this response.
1 points
Save Answer
Question 12
Mr Espent 65% of his salary on household expenses, and 15% of the remainder on travelling expenses and was finally left with R9 500. How much was his salary?​

Answers

Answer:

rs.1680.67

Step-by-step explanation:

His salary = x

remaining % = 100 - 65 = 35%

= 100 - 15 = 85%

x × 35/100 × 85/100 = 500

x = 1680.67

The ice cream man just ended his shift for the day. Let 1/2x^2 6/11x + 8 represent the amount of chocolate ice cream bars he sold. Let 5/9x^2 + 2/3 represent the amount of vanilla ice cream bars he sold. Finally let 1/3x^2 + 4x + 4/3 represent the amount of strawberry ice cream bars he sold. Select all the statements that are true

a. The total amount of ice cream bars sold can be represented by the expression 25/18x^2+ 50/11x +10
b. The total amount of ice cream bars sold can be represented by the expression 25/18x^2 + 172/33x +28/3
c. He sold 1/6x^2 + 50/11x + 28/3 more chocolate than strawberry ice cream bars.
d. He sold 1/6x^2 - 38/11x + 20/3 more chocolate than strawberry ice cream bars.

Answers

Answer:

A and D

Step-by-step explanation:

Total ice cream bars sold = sum of chocolate sold , vanilla and strawberry ice-creams sold.

=(1/2)x2 + (6/11)x + 8 + (5/9)x2 + (2/3) +(1/3)x2 + 4x +(4/3) (Given in the question)

=(25/18)x2 + (50/11)x + 10 (Adding terms corresponding to x2,x ,constant respectively)

Difference in chocolate and strawberry bars =[ (1/2)x2 + (6/11)x + 8] - [(1/3)x2 + 4x +(4/3)]

= (1/6)x2 - (38/11)x +(20/3)

So, the correct options are A and D

Hi, could someone help me solve this. so the question says to find the area of the shaded part (in black) , in terms of pie (π). the length of the square is 12 cm. the radius of the circle is 6cm. i came with the answer of (144-36π)/4. is this ok? below is the picture of the question. ​

Answers

Answer:

yes but can be simplified

Step-by-step explanation:

area of shaded part = ( area of square - area of circle ) / 4

= [tex]\frac{12^2-\pi (6)^2}{4}[/tex]

= [tex]\frac{144-36\pi }{4}[/tex]

= [tex]\frac{144}{4}[/tex] - [tex]\frac{36\pi }{4}[/tex]

= 36 - 9π

21(2-y)+12y=44 find y​

Answers

Answer: y= -2/9
Explanation:
21(2-y)+12y=44
42-21y+12y=44
42-9y=44
-9y=2
y=-2/9

Answer:

[tex]\textbf{HELLO!!}[/tex]

[tex]21\left(2-y\right)+12y=44[/tex]

[tex]42-21y+12y=44[/tex]

[tex]~add ~similar\:elements[/tex]

[tex]42-9y=44[/tex]

[tex]Subtract~42~from~both~sides[/tex]

[tex]42-9y-42=44-42[/tex]

[tex]-9y=2[/tex]

[tex]Divide\:both\:sides\:by\:}-9[/tex]

[tex]\frac{-9y}{-9}=\frac{2}{-9}[/tex]

[tex]y=-\frac{2}{9}[/tex]

----------------------

hope it helps...

have a great day!

Find the area of the following shape:

Answers

Answer:

36cm^2

Step-by-step explanation:

total area: 6x(4+3)=42

total area excluding the space: 42-(2x3)=36

Answer:

36 cm squared

Step-by-step explanation:

To solve this problem, I first construct a line. (shown in yellow in the first photo)

I then find the area of the top rectangle. (6 cm * 4 cm = 24 cm squared.)

Next, I find the area of the lower rectangle. But...to do that I have to find the length of the line that I constructed. To do this, I do  6cm-2cm=4cm.

Then I can find the area of the lower rectangle. (4cm*3cm=12cm squared.)

add up the area of both of the rectangles and.........12+24=36 cm squared

HELP ME PLEASE IF YOU DO YOU WILL GET BRAINLESS AND PLEASE EXPLAIN THE BEST YOU CAN

Answers

Answer:

<3=75°

Step-by-step explanation:

Angle 3 and angle 2x+95 are supplementary( supplementary angles add up to 180°)

So <3+2x+95=180

<3+2x=180-95

<3+2x=85( let's call this equation 1)

Next, angle 5 and angle 8x+71 are opposite angles (opposite angles are equal) therefore <5=8x+71

Now, <3 and <5 are co-interior angles(co-interior angles are supplementary)

So <3+8x+71=180

<3+8x=180-71=109

Thus, <3+8x=109(let's call this equation 2)

Now solving equation 1 and 2 simultaneously:

Make <3 the subject of equation 1

<3=85-2x

Put <3=85-2x into equation 2

85-2x+8x=109

6x=24

x=24/6=4

Now, remember that angle 2x+95 becomes

2(4)+95

8+95=103°

Therefore<3=180-105=75°

 Marsha has a bag that contains 4 green marbles, 8 yellow marbles , and 20 red marbles . If she chooses one marble from the bag, what is the probability that the marble is not red?

PLEASE HELP IF YOURE GOOD AT GEOMETRY!!

Answers

Answer:

C. 3/8

HOPE THIS HELPS :)

Answer:

c. 3/8

Step-by-step explanation:

first you need the denomerator by adding all marbles together which equals 32. now for the munerator you need the sum of the green and yellow marbles. this equals 12. so your fraction is 12/32. next we simplify. we can divide both numbers by 4. getting us a fraction of 3/8.

he speeds (in MPH) of automobiles traveling in a city are given below:
20, 35, 42, 52, 65, 49, 24, 37, 23, 41, 50, 58
The mean speed of the cars is

Answers

Answer:

Mean speed = 41.3 mph

Step-by-step explanation:

Given that,

The speeds of an automobiles are given below:

20, 35, 42, 52, 65, 49, 24, 37, 23, 41, 50, 58

We need to find the mean speed of the cars.

Mean = sum of observations/ no. of observation

[tex]M=\dfrac{20+35+42+52+65+49+24+37+23+41+50+58}{12}\\\\M=41.3[/tex]

So, the mean speed of the cars is equal to 41.3 mph.

Joe drives for 3 hours and covers 201 miles. In miles per hour, how fast was he driving?​

Answers

Answer:

67 mph

Step-by-step explanation:

201/3 = 67

Can you solve this problem

Answers

Answer:

x = 18

Step-by-step explanation:

8 x 18 - 3 = 141

The answer is to your question is 18

Solve the equation 2x^2 + 3 – 41 = –15 to the nearest tenth.

Hellllpppp

Answers

9514 1404 393

Answer:

 x = {-4.4, +2.9}

Step-by-step explanation:

We assume you want to solve ...

  2x^2 +3x -41 = -15

Adding 41 and factoring out the leading coefficient gives ...

  2(x^2 +3/2x) = 26

Dividing by 2 makes it ...

  x^2 +3/2x = 13

We can add the square of half the x-coefficient to "complete the square."

  x^2 +3/2x +(3/4)^2 = 13 +(3/4)^2

  (x +3/4)^2 = 13.5625 . . . . write the left side as a square

  x +3/4 = ±√13.5625 . . . . . take the square root

  x = -0.75 ±3.683 = {-4.433, +2.933} . . . . subtract 3/4 and evaluate

The solutions are approximately x = -4.4 and x = 2.9.

to train for a race, you plan to run 1 mile the first week and double the number of miles each week for five weeks. How many miles will you run for the 5th week. math problem

Answers

Answer:

16 Miles

Step-by-step explanation:

For every week you simply multiply the number of miles from the previous week by 2, therefore

Week 1: 1

Week 2: 2

Week 3: 4

Week 4: 8

Week 5: 16

A person walks 1/6 mile in 1/18 hour.

The person's speed is _ miles per hour.

Answers

This Is What I Got!

Hope This Helps! :)

Have A Good Day!!

And If You Can I Wouldn't Mind A Brainliest! :))

Answer:

Divide 1/6 miles to 1/12hour since u wanna find our miles per hour

So it’ll be : 1/6 / 1/12

= 1/6 x 12/1

= 2 miles

Find the distance between the pair of points: (0,1) and (1,0)

Answers

Answer:

sqrt(1^2 + 2^2)

[tex]\sqrt{2}[/tex]

Step-by-step explanation:

xác định m để 3 vector sau đây phụ thuộc tuyến tính
u=(m+1,1,m+1),v=(1,1,1),u(2,0,m+2



0

Answers

Answer:

i dont know hahaha

Step-by-step explanation:

soryy

If using the method of completing the square to solve the quadratic equation x^2+15x+21=0, which number would have to be added to "complete the square"?

Answers

Step-by-step explanation:

the answer is in the above image

Answer:

my answer is 225/5 sorry comments for wrong

Other Questions
Given the equation 4square root of x minus 3 = -12, solve for x and identify if it is an extraneous solution.A. x = 0, solution is extraneousB. x = 0, solution is not extraneousC. x = 12, solution is extraneousD. x = 12, solution is not extraneous Which expression is equivalent to 10k + 17 - 7j - 18 - 11k?-8jk - 1-7j - k - 1-7j + k + 1-8j - kPLEASE HURRY !!!!20 POINTS!!! Answer this fast and correct and you will get a thanks, a brainliest and a 5-star review along with 10 points! The exponent on b when b^3 is multiplied by b^3 isA. 3B. 6C. 9 Hi Everyone hope u all r doin well Pls Answer What happens when dilute sulphuric acid is poured on a copper plate ? Indicate the equation of the given line in standard form, writing the answer in the equation box below.The line containing the longer diagonal of a quadrilateral whose vertices are A(2, 2), B(-2,-2), C(1, -1), and D16, 4). A balloon is filled with 80 liters of gas on a day where the temperature was 34 degrees at sea level which is 101.3 kPa and released. As the balloon rises to a certain altitude, the temperature drops to 0 degrees celsius and the balloon doubles in volume. What is the atmospheric pressure at that altitude? A crucible (container) of molten metal has an open top with an area of 5.000 m^2. The molten metal acts as a blackbody radiator. The intensity spectrum of its radiation peaks at a wavelength of 320 nm. What is the temperature of that blackbody? Find the distance between the pair of points: (0,-8)and (0,4) One of the biggest challenges that we all face, at least inmy opinion it's a challenge, is making difficult decisions,life-altering decisions. These decisions are not onlydifficult to make, but they can also bring consequencesthat are not easy to live with. However, if decisions are wellthought out and carefully considered they will be easier tomake and easier to live with.What feedback would be most helpful feedback to give the writer of thisparagraph?O A. Make the claim more personal.B. Address a counterclaim.O C. Revise the claim to make it clearer.D. Improve the spelling and grammar. Your English teacher asks you to write a story. You have to start with the story: I had a natural occurrence when I turned on the television. PLEASE HELP WILL MARK BRAINLIEST - An airplane travels 150 miles horizontally during a decrease of 35,000 feet vertically.1. What is the angle of descent?2. How long is the plane's path? What are the rational roots of f(d) = 5d - 6 + d-8? When curare, a neuromuscular poison, is dropped onto an isolated muscle-nerve preparation in a laboratory, the muscle does not contract when the nerve is stimulated, even though neurotransmitter is released from the nerve cell. Why does this happen Which of the following techniques employs a probe consisting of a labeled, single-stranded DNA of known sequence that can find and bind to a complementary sequence present in a large sample of different DNA sequences?a. DNA hybridizationb. RFLPc. PCRd. DNA sequencing What is the function of histamine in inflammation(Will mark as brainliest) is a 6.33 a D grade or a C grade What are the two largest religions in the world today?O A. Hinduism and ChristianityO B. Islam and HinduismO C. Christianity and BuddhismD. Christianity and Islam PLSSS, NEED ANSWER. Find the midpoint of the line segment with end coordinates of (-2,-5 and 3,-2). Give coordinates as decimals where apropriate please tell me how a guy who is quote on quote "in love with you" can't wait till you get out of lockup to "love" you and has to get someone else???????